You are on page 1of 12

Math 2260 Exam #3 Practice Problem Solutions

1. Does the following series converge or diverge? Explain your answer.



X 2n
.
n=0
3n + n3

Answer: Since 3n + n3 > 3n for all n ≥ 1, it follows that


 n
2n 2n 2
< = .
3n + n3 3n 3

Therefore,
∞ ∞  n
X 2n X 2 1
n 3
< = 2 = 3.
n=0
3 +n n=0
3 1− 3

Hence, the given series converges.


2. Does the following series converge or diverge? Explain your answer.

X n
n
.
n=1
3

Answer: Use the Ratio Test:


n+1
3n+1 n + 1 3n 1 n+1 1
lim = lim · = lim · = .
n→∞ nn n→∞ 3n+1 n n→∞ 3 n 3
3

1
Since 3 < 1, the Ratio Test implies that this series converges.
3. Does the following series converge or diverge? Explain your answer.
∞  
X 1
2n sin .
n=1
n

Answer: Notice that the terms of this series are not going to zero:
   
1 1
lim 2n sin = lim 2x sin
n→∞ n x→∞ x
sin x1

= lim 1x→∞
2x
1 −2

cosx · x2
= lim −2
x→∞
(2x)2

−2 cos x1

4x2
= lim ·
x→∞ x2 −2
 
1
= lim 4 cos
x→∞ x
=4

where I went from the second to the third lines using L’Hôpital’s Rule. Since the limit of the terms is
equal to 4, not zero, the series must diverge.

1
4. Does the following series converge or diverge? If it converges, find the sum. If it diverges, explain why.

X 2n + 3 n
.
n=1
4n

Answer: Re-writing slightly, the given series is equal to


∞  n ∞ ∞
3n 2n X 3n
 X
X 2
+ = + .
n=1
4n 4n n=1
4n n=1 4n

Since both of these series are convergent geometric series, I know the original series converges, so it
remains only to determine the sum. Notice that
∞ ∞  n−1
2n
  X
X 2 4 8 2 2 4 2 2 2/4 1/2
n
= + + + . . . = 1 + + + . . . = = = = 1.
n=1
4 4 16 64 4 4 16 n=1
4 4 1 − 2/4 1/2

Similarly,
∞ ∞  n−1
3n
  X
X 3 9 27 3 3 9 3 3 3/4 3/4
n
= + + + ... = 1+ + + ... = = = = 3.
n=1
4 4 16 64 4 4 16 n=1
4 4 1 − 3/4 1/4

Therefore,
∞ ∞ ∞
X 2n + 3n X 2n X 3n
= + = 1 + 3 = 4.
n=1
4n n=1
4n n=1 4n

5. Find the interval of convergence of the power series



X (2x − 5)n
.
n=1
n2 3 n

Answer: We use the Ratio Test on the series of absolute values to first determine the radius of
convergence:

(2x−5)n+1
(n+1)2 3n+1 |2x − 5|n+1 n3n |2x − 5| n2 |2x − 5|
lim = lim
2 n+1
· n
= lim · 2
= .
n→∞ n
(2x−5)
n2 3n n→∞ (n + 1) 3 |2x − 5| n→∞ 3 (n + 1) 3

|2x−5|
Therefore, the given series converges absolutely when 3 < 1, meaning when |2x − 5| < 3.
Now we check the endpoints. When 2x − 5 = 3, the series becomes
∞ ∞
X 3n X 1
2 3n
= 2
,
n=1
n n=1
n

which converges.
Likewise, when 2x − 5 = −3, then series becomes
∞ ∞ ∞
X (−3)n X (−1)n 3n X (−1)n
= = ,
n=1
n2 3 n n=1
n2 3 n n=1
n2

which also converges.


Therefore, the series converges for all x so that
−3 ≤ 2x − 5 ≤ 3,
which is the interval [1, 4].

2
6. Use the first two non-zero terms of an appropriate Taylor series to approximate
Z 1
sin(x2 ) dx.
0

Estimate the error of your approximation (i.e. the difference between your answer and the actual value
of the integral).
Answer: First, recall that the Taylor series centered at x = 0 for sin(x) is

x3 x5 x7
sin(x) = x − + − + ....
3! 5! 7!
Therefore, the Taylor series centered at x = 0 for sin(x2 ) is

(x2 )3 (x2 )5 (x2 )7 x6 x10 x14


sin(x2 ) = (x2 ) − + − + . . . = x2 − + − + ....
3! 5! 7! 3! 5! 7!

Hence,
1 1 1
x6 x10
 3
x7 x11
Z Z  
x 1 1 1
sin(x2 ) dx = x2 − + − . . . dx = − + − ... = − + −. . . .
0 0 3! 5! 3 7 · 3! 11 · 5! 0 3 42 11 · 5!

Therefore, we can approximate this number by


1 1 13
− = ,
3 42 42
and we know the error is no bigger than
1 1 1
= = ,
11 · 5! 11 · 120 1320
so in particular the estimate 13
42 is accurate to 3 decimal places. (It would be totally fine in an exam
1
situation to leave your answer as 11·5! .)

7. Find the radius of convergence of the Taylor series


∞ √
X (−1)n 1 + n
2
(x − 2)n .
n=2
n

Answer: Use the Ratio Test on the series of absolute values:


(−1)n+1 √1+(1+n)



(n+1)2 (x − 2)n+1 √
2 + n |x − 2|n+1 n2
lim √ = lim · √
n→∞ n
(−1) 1+n
(x − 2)n
n→∞ (n + 1)2 1 + n |x − 2|n
n2

2+n n2
= lim |x − 2| · √ ·
n→∞ 1 + n (n + 1)2
= |x − 2|.

Therefore, the series converges absolutely when |x − 2| < 1, so the radius of convergence is equal to 1.

3

8. Write the second-degree
√ Taylor polynomial for f (x) = x centered at c = 25. Use this polynomial to
approximate 26 and estimate the error of this approximation.

Answer: I can write down the Taylor series centered at x = 25 for the function f (x) = x by first
computing the derivatives of f :
1 −1/2 1
f 0 (x) = x = √
2 2 x
1 −1 −3/2 −1
f 00 (x) = · x = 3/2
2 2 4x
000 −1 −3 −5/2 3
f (x) = · x = 5/2
4 2 8x
Therefore,

f (25) =
25 = 5
1 1
f 0 (25) = √ =
2 25 10
−1 −1 −1
f 00 (25) = = =
4 · 253/2 4 · 125 500
3 3 3
f 000 (25) = = =
8 · 255/2 8 · 3125 25, 000

Hence, the Taylor series centered at x = 25 for x is

√ (x − 25) (x − 25)2 3(x − 25)3


x=5+ − + − ...
10 1000 150, 000
Therefore,
√ (26 − 25) (26 − 25)2 1 1
26 ≈ 5 + − =5+ − = 5.1 − 0.001 = 5.099,
10 1000 10 1000
with an error no more than
3(26 − 25)3 3 1
= = = 0.00002.
150, 000 150, 000 50, 000

(In fact, 26 ≈ 5.0990195.)
9. Does the series
∞ √
X n
n
n 2
n=1

converge or diverge. Be sure to give a complete explanation.


√ P 1
Answer: Since limn→∞ n n = 1 as we discussed in class, a limit comparison to the series n2 is a
natural: √
n n √
n2
n
n n2 √
lim 1 = lim 2
· = lim n n = 1.
n→∞
n2
n→∞ n 1 n→∞
P 1
Therefore, since the series n2 converges, the Limit Comparison Test implies that the given series
converges as well.

4
10. Does the following series converge or diverge? Explain your answer.

X n!(n + 1)!
.
n=1
(3n)!

Answer: Use the Ratio Test:


(n+1)!(n+2)!
(3(n+1))! (n + 1)!(n + 2)! (3n)! (n + 1)(n + 2)
lim n!(n+1)!
= lim · = lim = 0,
n→∞ n→∞ (3n + 3)! n!(n + 1)! n→∞ (3n + 3)(3n + 2)(3n + 1)
(3n)!

since the numerator is a polynomial of degree 2 but the denominator is a polynomial of degree 3.
Therefore, since 0 < 1 the Ratio Test implies that the series converges.
11. Does the sequence ∞
n2
 
arctan
n2 + 1 n=1
converge or diverge? If it converges, find the limit; if it diverges, explain why.
Answer: First, notice that
n2
lim = 1.
n→∞ n2 + 1

Therefore, the term inside the arctangent is going to 1, so

n2
 
π
lim arctan = arctan(1) = .
n→∞ n2 + 1 4

12. Does the series



X 1
2−

n=2
n n
converge or diverge? Explain your answer.
2

Answer:P 1For large n, the n should dominate the n, so let’s do a limit comparison to the convergent
series n2 .

1√
n2 − n 1 n2 n2 1
lim 1 = lim √ · = lim √ = lim 1 = 1.
n→∞ n→∞ n2 − n 1 n→∞ 2
n − n n→∞ 1 − n3/2
n2

1 1√
P P
Therefore, since n2 converges, so does n2 − n
.

13. Does the series



X n!
n=1
n5
converge or diverge? Explain your answer.
Answer: Use the Ratio Test:
(n+1)!
(n+1)5 (n + 1)! n5 n5
lim n!
= lim · = lim (n + 1) =∞
n→∞ n→∞ (n + 1)5 n! n→∞ (n + 1)5
n5

n5
since the expression (n+1)5 is going to 1 but (n + 1) is going to ∞.
Therefore, the Ratio Test implies that the series diverges.

5
14. Does the series

X 3n
n=1
n3
converge or diverge? Explain your answer.
Answer: Use the Ratio Test:
3n+1
(n+1)3 3n+1 n3 (n + 1)3
lim 3n = lim · = lim 3 · = 3.
n→∞ n→∞ (n + 1)3 3n n→∞ n3
n3

Since 3 > 1, the Ratio Test implies that the series diverges.

15. Does the series



X 2n + 3
n=0
(n2 + 3n + 6)2
converge or diverge? Explain your answer.
Answer: For n very large,P the denominator will be dominated by the term n4 , so do a limit comparison
n
to the convergent series n4 :

2n+3
(n2 +3n+6)2 2n + 3 n4 2n + 3 n4
lim n = lim · = lim · = 2 · 1 = 2.
n→∞ n→∞ (n2 + 3n + 6)2 n n→∞ n (n2 + 3n + 6)2
n4

n 1
P
Therefore, since the limit is finite and the series n4 = n3 converges, the Limit Comparison Test
implies that the given series converges as well.

16. For which values of x does the series



X (x − 4)n
n=0
5n
converge? What is the sum of the series when it converges?
Answer: First, use the Ratio Test on the series of absolute values:

(x−4)n+1
5n+1 |x − 4|n+1 5n |x − 4|
lim = lim n+1
· n
= ,
n→∞ (x−4)n
5n n→∞ 5 |x − 4| 5

so the given series converges absolutely whenever |x−4| 5 < 1, meaning when |x − 4| < 5 (from this we
see that the radius of convergence of the series is 5).
Now check the endpoints. When x − 4 = 5, the series becomes
∞ ∞
X 5n X
= 1,
n=0
5n n=0

which diverges.
Similarly, when x − 4 = −5, the series becomes
∞ ∞ ∞
X (−5)n X (−1)n 5n X
n
= n
= (−1)n ,
n=0
5 n=0
5 n=0

which also diverges.

6
Therefore, the series converges for
−5 < x − 4 < 5,
which is to say, on the interval (−1, 9).
When the series does converge, it is just the geometric series
X  x − 4 n 1 1 5
= x−4 = 5−(x−4) = ,
n=0
5 1− 5 9−x
5

5
so when it converges the series converges to the function f (x) = 9−x .

17. Does the series



X (−1)n (n2 + n3 )
n=1
n4 + 1
converge absolutely, converge conditionally, or diverge? Explain your answer.
Answer: The series of absolute values
∞ ∞
(−1)n (n2 + n3 ) X n2 + n3
X
=
n=1
n4 + 1
n=1
n4 + 1
P n3
should behave similarly to n4 , so do a limit comparison to this series:
n2 +n3
n4 n2 + n3 n4 n2 + n3
lim n3
= lim 4
· 3 = lim = 1.
n→∞
n4
n→∞ n n n→∞ n3
P n3 P1
Therefore, since n4 = n diverges, the Limit Comparison Test says that the series of absolute
values diverges as well.
However, the given series satisfies the hypotheses of the Alternating Series Test, so it converges. There-
fore, since the series converges but the series of absolute values diverges, we conclude that the series
converges conditionally.
18. Does the series

X (2n)!
2n (n!)2
n=1

converge or diverge? Explain your answer.


Answer: Use the Ratio Test:
(2(n+1))!
2n+1 ((n+1)!)2 (2n + 2)! 2n (n!)1
lim = lim ·
n→∞ (2n)! n→∞ 2n+1 ((n + 1)!)2 (2n)!
2n (n!)2
(2n + 2)(2n + 1) (n!)2
= lim ·
n→∞ 2 ((n + 1)n!)2
(2n + 2)(2n + 1) 1
= lim · .
n→∞ 2 (n + 1)2

Since (2n + 2)(2n + 1) = 4n2 + 6n + 2 and since (n + 1)2 = n2 + 2n + 1, the above limit is equal to

4n2 + 6n + 2 4
lim = = 2.
n→∞ 2(n2 + 2n + 1) 2

Since 2 > 1, the Ratio Test implies that the given series diverges.

7
19. Does the series

X (−1)n
n=2
n ln(n)
converge absolutely, converge conditionally, or diverge? Explain your answer.
Answer: The series of absolute values
∞ ∞
(−1)n X

X
= 1
n ln(n) n ln(n)
n=2 n=2

diverges (see HW #12, Problem 3 for a proof of this). However, the series satisfies the hypotheses of
the Alternating Series Test and hence converges, so we see that it converges conditionally.
20. For what values of p does the series

X 1
(n 2 + 1)p
n=0

converge? Explain your answer.


1
P
Answer: For large n, the +1 will barely contribute, so do a limit comparison with the series (n2 )p :

1 p
(n2 )p n2

(n2 +1)p 1
lim 1 = lim 2 p
· = lim 2
= 1p = 1,
n→∞ n→∞ (n + 1) 1 n→∞ n +1
(n2 )p

n2
since limn→∞ n2 +1 = 1.
P 1 P 1
Therefore, the given series and the series (n2 )p = n2p will either both converge or both diverge.
P 1
Since n2p converges for 2p > 1 and diverges otherwise, we see that the given series converges when
2p > 1, which is to say when
1
p> .
2
21. What is the interval of convergence of the following power series? Explain your answer.

X 3n (x − 2)n
.
n=1
n2

Answer: Start by applying the Ratio Test to the series of absolute values:
n+1
3 (x−2)n+1
(n+1)2 3n+1 |x − 2|n+1 n2 n2
lim n = lim · = lim 3|x − 2| · = 3|x − 2|.
n→∞ 3 (x−2)n
n2 n→∞ (n + 1)2 3n |x − 2|n n→∞ (n + 1)2

Therefore, the ratio test implies that the given series converges absolutely when 3|x − 2| < 1, meaning
when |x − 2| < 31 .
Now, check the endpoints. When x − 2 = 13 , the series becomes
∞ 1 n ∞

X 3n 3
X 1
= ,
n=1
n2 n=1
n 2

which converges.

8
Likewise, when x − 2 = − 13 , the series becomes
∞ n ∞
X 3n − 31 X (−1)n
= ,
n=1
n2 n=1
n2

which also converges.


Therefore, the series converges for
1 1
− ≤x−2≤ ,
3 3
5 7
meaning the interval of convergence is 3 , 3 .
22. What are the first four nonzero terms of the Taylor series centered at x = 0 for the function f (x) = xe3x ?
Answer: Since
x2 x3
ex = 1 + x + + + ...,
2! 3!
we see that
(3x)2 (3x)3 9x2 27x3
e3x = 1 + (3x) + + + . . . = 1 + 3x + + + ....
2! 3! 2! 3!
Therefore,
9x2 27x3 9x3 27x4
 
3x
xe = x 1 + 3x + + + . . . = x + 3x2 + + + ....
2! 3! 2! 3!
R 1/2
23. Which of the following gives the value of 0
cos(x2 ) dx correct to within 0.0001 (i.e. within 1/10, 000)?
Explain your answer.
1 1 1 1 1 1 1 1 1 1 1
(a) (b) − (c) − (d) − (e) − (f) −
2 2 120 24 2688 2 320 2 384 24 1024

Answer: Since
x2 x4
cos(x) = 1 − + − ...,
2! 4!
the Taylor series for cos(x2 ) centered at x = 0 is

(x2 )2 (x2 )4 x4 x8
cos(x2 ) = 1 − + − ... = 1 − + − ....
2! 4! 2 24
Therefore,
1/2 1/2
x4 x8
Z Z  
cos(x2 ) dx = + 1− − . . . dx
0 0 2 24
 5 9
1/2
x x
= x− + − ...
10 9 · 24 0
1 1 1
= − + − ....
2 320 9 · 24 · 29
Hence, choice (d) is clearly the best approximation.
24. What is the limit of the following sequence? Explain your answer.
 ∞
sin(arctan(ln(n)))
n=2

9
Answer: Since limn→∞ ln(n) = +∞ and since limx→π/2+ tan(x) = +∞, it follows that
π
lim arctan(ln(n)) = .
n→∞ 2

Therefore,
lim sin(arctan(ln(n))) = sin(π/2) = 1.
n→∞

25. Does the series √



X n
n=2
n2 −n+1
converge or diverge? Explain your answer.
Answer: For large n, we should expect that n2 will dominate the denominator, so do a limit comparison
P √n
to n2 : √
n √
n2 −n+1 n n2 n2
lim √ = lim · √ = lim = 1.
n→∞ n n→∞ n2 − n + 1 n n→∞ n2 − n + 1
n2

P √n P 1
Therefore, since the series n2 = n3/2
converges, so does the given series.
26. Does the series

X (−1)n (n2 + n3 )
n=1
n4 + ln(n)
converge absolutely, converge conditionally, or diverge? Explain your answer.
Answer: First, notice that the series satisfies the Hypotheses of the Alternating Series Test, so it
definitely converges. To see whether it converges absolutely, consider the series of absolute values
∞ ∞
(−1)n (n2 + n3 ) X n2 + n3

X
= .
n4 + ln(n) n4 + ln(n)
n=1 n=1

Now, you should expect the n3 to dominate the numerator and the n4 to dominate the denominator,
P n3
so do a limit comparison to the series n4 :

n2 +n3
n4 +ln(n) n2 + n3 n4
lim n3
= lim 4
· 3
n→∞ n→∞ n + ln(n) n
n4
n2 + n3 n4
= lim ·
n→∞ n3 n4 + ln(n)
1
+1 1
= lim n ·
n→∞ 1 1 + ln(n)
4 n
=1·1
= 1.

P n3 P1
Therefore, since the series n4 = n diverges, the Limit Comparison Test implies that the series of
absolute values diverges, and hence the given series only converges conditionally.
27. If it converges, find the sum of the series

X (−1)n
.
n=0
n!2n

10
If the series diverges, explain why.
Answer: This is tricky. Remember that

x2 x3 X xn
ex = 1 + x + + + ... = .
2! 3! n=0
n!

But now if I let x = −1/2 on the right hand side, I get the series
∞ ∞
X (−1/2)n X (−1)n
= ,
n=0
n! n=0
n!2n

which is the given series. Hence, the given series sums to


1
e−1/2 = √ .
e

28. For which values of p does the following series converge? Explain your answer.

X √
1 + np .
n=1

Answer: Notice that for any p, 1 + np > 1, meaning that


√ √
1 + np > 1 = 1.

Therefore,
∞ ∞
X √ X
1 + np > 1,
n=1 n=1

which obviously diverges, so the given series also diverges.


29. What is the interval of convergence of the following series? Explain your answer.

X (x − 1)n
.
n=1
n2

Answer: First, use the Ratio Test on the series of absolute values:

(x−1)n+1
(n+1)2 |x − 1|n+1 n2 n2
lim = lim
2
· n
= lim |x − 1| · = |x − 1|.
n→∞ n
(x−1)
n2 n→∞ (n + 1) |x − 1| n→∞ (n + 1)2

Thus, the Ratio Test says that the given series converges absolutely when |x − 1| < 1.
Now check the endpoints. When x − 1 = 1, the series becomes
∞ ∞
X 1n X 1
2
= ,
n=1
n n=1
n2

which converges.
Likewise, when x − 1 = −1, the series becomes

X (−1)n
,
n=1
n2

11
which also converges.
Therefore, the given series converges for

−1 ≤ x − 1 ≤ 1,

so the interval of convergence is [0, 2].

12

You might also like